Skip to main content

Theory and Modern Applications

On the solutions of a max-type system of difference equations of higher order

Abstract

In this paper, we study the following max-type system of difference equations of higher order:

$$ \textstyle\begin{cases} x_{n} = \max \{A ,\frac{y_{n-t}}{x_{n-s}} \}, \\ y_{n} = \max \{B ,\frac{x_{n-t}}{y_{n-s}} \},\end{cases}\displaystyle \quad n\in \{0,1,2,\ldots \}, $$

where \(A,B\in (0, +\infty )\), \(t,s\in \{1,2,\ldots \}\) with \(\gcd (s,t)=1\), the initial values \(x_{-d},y_{-d},x_{-d+1},y_{-d+1}, \ldots , x_{-1}, y_{-1}\in (0,+ \infty )\) and \(d=\max \{t,s\}\).

1 Introduction

Concrete nonlinear difference equations and systems have attracted some recent attention (see, e.g., [139]). One of the classes of such equations/systems are max-type difference equations/systems. For some results of solutions of many max-type difference equations and systems, such as eventual periodicity, the boundedness character and attractivity, see, e.g. [15, 79, 1116, 1825, 2830, 3236, 38, 39] and the references therein. Our purpose in this paper is to study the eventual periodicity of the following max-type system of difference equation of higher order:

$$ \textstyle\begin{cases} x_{n} = \max \{A ,\frac{y_{n-t}}{x_{n-s}} \}, \\ y_{n} = \max \{B ,\frac{x_{n-t}}{y_{n-s}} \},\end{cases}\displaystyle \quad n\in { \mathbf{{N}}}_{0}\equiv \{0,1,\ldots \}, $$
(1.1)

where \(A,B\in {\mathbf{{R}}}_{+}\equiv (0,+\infty )\), \(t,s\in {\mathbf{{N}\equiv }\{\mathbf{1},\mathbf{2},\ldots \}}\) with \(\gcd (s,t)=1\), the initial values \(x_{-d},y_{-d}, x_{-d+1},y_{-d+1}, \ldots , x_{-1}, y_{-1}\in \mathbf{{R}}_{+}\) and \(d=\max \{t,s\}\).

When \(t=1\) and \(s=2\), (1.1) reduces to the max-type system of difference equations

$$ \textstyle\begin{cases} x_{n} = \max \{A ,\frac{y_{n-1}}{x_{n-2}} \}, \\ y_{n} = \max \{B ,\frac{x_{n-1}}{y_{n-2}} \},\end{cases}\displaystyle \quad n\in { \mathbf{{N}}}_{0}. $$
(1.2)

Fotiades and Papaschinopoulos in [5] showed that every positive solution of (1.2) is eventually periodic.

In 2012, Stević [23] obtained in an elegant way the general solution to the following max-type system of difference equations:

$$ \textstyle\begin{cases} x_{n+1}=\max \{\frac{A}{x_{n}},\frac{y_{n}}{x_{n}} \}, \\ y_{n+1}=\max \{\frac{A}{y_{n}},\frac{x_{n}}{y_{n}} \},\end{cases}\displaystyle \quad n\in \mathbf{{N}}_{0}, $$
(1.3)

for the case \(x_{0},y_{0}\geq A>0\) and \(y_{0}/x_{0}\geq \max \{A,1/A\}\).

In [35], Sun and Xi studied the following max-type system of difference equations:

$$ \textstyle\begin{cases} x_{n} = \max \{\frac{1}{x_{n-m}} , \min \{1,\frac{A}{y_{n-r}} \} \}, \\ y_{n} = \max \{\frac{1}{y_{n-m}} , \min \{1,\frac{B}{x_{n-t}} \} \},\end{cases}\displaystyle \quad n\in { \mathbf{{N}}}_{0}, $$
(1.4)

where \(A,B\in { \mathbf{{R}}}_{+}\), \(m,r,t\in {\mathbf{{N}}}\) and the initial values \(x_{-d},y_{-d},x_{-d+1},y_{-d+1}, \ldots , x_{-1}, y_{-1}\in {\mathbf{{R}}}_{+}\) with \(d=\max \{m,r,t\}\) and showed that every positive solution of (1.4) is eventually periodic with period 2m.

When \(m=r=t=1\) and \(A=B\), (1.4) reduces to the max-type system of difference equations

$$ \textstyle\begin{cases} x_{n}=\max \{\frac{1}{x_{n-1}},\min \{1,\frac{A}{y_{n-1}} \} \}, \\ y_{n}=\max \{\frac{1}{y_{n-1}},\min \{1,\frac{A}{x_{n-1}} \} \}, \end{cases}\displaystyle \quad n\in \mathbf{{N}}_{0}. $$
(1.5)

Yazlik et al. [39] in 2015 obtained in an elegant way the general solution of (1.5).

In 2012, Stević [24] studied the following max-type system of difference equations:

$$ \textstyle\begin{cases} y^{(1)}_{n}=\max_{1\leq i\leq m_{1}} \{f_{1i}(y^{(1)}_{n-k_{i,1}^{(1)}},y^{(2)}_{n-k_{i,2}^{(1)}}, \ldots ,y^{(l)}_{n-k_{i,l}^{(1)}},n),y_{n-s}^{(1)} \}, \\ y^{(2)}_{n}=\max_{1\leq i\leq m_{2}} \{f_{2i}(y^{(1)}_{n-k_{i,1}^{(2)}},y^{(2)}_{n-k_{i,2}^{(2)}}, \ldots ,y^{(l)}_{n-k_{i,l}^{(2)}},n),y_{n-s}^{(2)} \}, \\ \ldots , \\ y^{(l)}_{n}=\max_{1\leq i\leq m_{l}} \{f_{li}(y^{(1)}_{n-k_{i,1}^{(l)}},y^{(2)}_{n-k_{i,2}^{(l)}}, \ldots ,y^{(l)}_{n-k_{i,l}^{(l)}},n),y_{n-s}^{(l)} \},\end{cases}\displaystyle \quad n\in \mathbf{{N}}_{0}, $$
(1.6)

where \(s,l,m_{j},k^{(j)}_{i,t}\in {\mathbf{{N}}}\) (\(j,t\in \{1,2,\ldots ,l \}\)) and \(f_{ji}:\mathbf{{R}}_{+}^{l}\times {\mathbf{{ N}}}_{0}\longrightarrow \mathbf{{R}}_{+}\) (\(j\in \{1,\ldots ,l\}\) and \(i\in \{1,\ldots ,m_{j}\}\)), and showed that every positive solution of (1.6) is eventually periodic with (not necessarily prime) period s if \(f_{ji}\) satisfy some conditions.

Moreover, Stević et al. [29] in 2014 investigated the following max-type system of difference equations:

$$ \textstyle\begin{cases} y^{(1)}_{n}=\max_{1\leq i_{1}\leq m_{1}} \{f_{1i_{1}}(y^{(1)}_{n-k_{i_{1},1}^{(1)}},y^{(2)}_{n-k_{i_{1},2}^{(1)}}, \ldots ,y^{(l)}_{n-k_{i_{1},l}^{(1)}},n),y_{n-t_{1}s}^{(\sigma (1))} \}, \\ x^{(2)}_{n}=\max_{1\leq i_{2}\leq m_{2}} \{f_{2i_{2}}(y^{(1)}_{n-k_{i_{2},1}^{(2)}},y^{(2)}_{n-k_{i_{2},2}^{(2)}}, \ldots ,y^{(l)}_{n-k_{i_{2},l}^{(2)}},n),y_{n-t_{2}s}^{(\sigma (2))} \}, \\ \ldots, \\ y^{(l)}_{n}=\max_{1\leq i_{l}\leq m_{l}} \{f_{li_{l}}(y^{(1)}_{n-k_{i_{l},1}^{(l)}},y^{(2)}_{n-k_{i_{l},2}^{(l)}}, \ldots ,y^{(l)}_{n-k_{i_{l},l}^{(l)}},n),y_{n-t_{l}s}^{(\sigma (l))} \},\end{cases}\displaystyle \quad n\in \mathbf{{N}}_{0}, $$
(1.7)

where \(s,l,m_{j},t_{j},k^{(j)}_{i_{j},h}\in { \mathbf{{N}}}\) (\(j,h\in \{1,2,\ldots ,l\}\)), \((\sigma (1),\ldots ,\sigma (l))\) is a permutation of \((1,\ldots ,l)\) and \(f_{ji_{j}}:\mathbf{{R}}_{+}^{l}\times {\mathbf{{ N}}}_{0}\longrightarrow \mathbf{{R}}_{+}\) (\(j\in \{1,\ldots ,l\}\) and \(i_{j}\in \{1,\ldots ,m_{j}\}\)). They showed that every positive solution of (1.7) is eventually periodic with period sT for some \(T\in \mathbf{{N}}\) if \(f_{ji_{j}}\) satisfy some conditions.

2 Main results and proofs

In this section, we study the eventual periodicity of positive solutions of system (1.1). Let \(\{(x_{n},y_{n})\}_{n\geq -d}\) be a solution of (1.1) with the initial values \(x_{-d},y_{-d},x_{-d+1},y_{-d+1},\ldots , x_{-1}, y_{-1}\in { \mathbf{{R}}}_{+}\).

Lemma 2.1

If\(x_{n}=A\)eventually, then\(y_{n}\)is a periodic sequence with period 2seventually. If\(y_{n}=B\)eventually, then\(x_{n}\)is a periodic sequence with period 2seventually.

Proof

Assume that \(x_{n}=A\) eventually. By (1.1) we see

$$ y_{n} = \max \biggl\{ B ,\frac{A}{y_{n-s}} \biggr\} \quad \mbox{eventually}, $$
(2.1)

which implies \(y_{n}y_{n-s}\geq A\) eventually and

$$ \begin{aligned}[b] B&\leq y_{n} = \max \biggl\{ B , \frac{A}{y_{n-s}} \biggr\} \\ &= \max \biggl\{ B ,\frac{Ay_{n-2s}}{y_{n-s}y_{n-2s}} \biggr\} \\ &\leq \max \{B,y_{n-2s}\}\leq y_{n-2s} \quad \mbox{eventually}.\end{aligned} $$
(2.2)

Then, for any \(0\leq i\leq 2s-1\), \(y_{2ns+i}\) is eventually nonincreasing.

We claim that, for every \(0\leq i\leq 2s-1\), \(y_{2ns+i}\) is a constant sequence eventually. Assume on the contrary that, for some \(0\leq i\leq 2s-1\), \(y_{2ns+i}\) is not a constant sequence eventually. Then there exists a sequence of positive integers \(k_{1}< k_{2}<\cdots \) such that, for any \(n\in { \mathbf{{N}}}\), we have

$$ \begin{aligned}[b] B&< y_{2sk_{n+1}+i}=\frac{A}{y_{2sk_{n+1}+i-s}} \\ &< y_{2sk_{n}+i}=\frac{A}{y_{2sk_{n}+i-s}},\end{aligned} $$
(2.3)

which implies \(y_{2sk_{n+1}+i-s}>y_{2sk_{n}+i-s}\) for any \(n\in { \mathbf{{N}}}\). This is a contradiction. Thus \(y_{n}\) is a periodic sequence with period 2s eventually. The second case follows from the previously proved one by interchanging letters. The proof is complete. □

Lemma 2.2

If\(A\geq B\geq 1/A\), then\(x_{2(n+1)t+i}\leq x_{2nt+i}\)for any\(n\geq t+s\)and\(i\in { {\mathbf{{N}}}}_{0}\). If\(B\geq A\geq 1/B\), then\(y_{2(n+1)t+i}\leq y_{2nt+i}\)for any\(n\geq t+s\)and\(i\in { {\mathbf{{N}}}}_{0}\).

Proof

Assume that \(A\geq B\geq 1/A\). By (1.1) we see that \(x_{n}\geq A\) and \(y_{n}\geq B\) for any \(n\in { \mathbf{{N}}}_{0}\), and

$$ x_{2(n+1)t+i} = \max \biggl\{ A,\frac{B}{x_{2(n+1)t+i-s}}, \frac{x_{2nt+i}}{x_{2(n+1)t+i-s}y_{2(n+1)t+i-t-s}} \biggr\} . $$
(2.4)

Since \(B/x_{2(n+1)t+i-s}\leq B/A\leq 1\leq A\) and \(x_{2(n+1)t+i-s}y_{2(n+1)t+i-t-s}\geq AB\geq 1\) for \(2(n+1)t+i\geq t+s\), we obtain

$$ x_{2(n+1)t+i}\leq \max \{A, x_{2nt+i}\}\leq x_{2nt+i}. $$
(2.5)

The second case follows from the previously proved one by interchanging letters. The proof is complete. □

Theorem 2.1

Let\(AB>1\). If\(A\geq B\), then\(x_{n}=A\)eventually and\(y_{n}\)is a periodic sequence with period 2seventually. If\(B> A\), then\(y_{n}=B\)eventually and\(x_{n}\)is a periodic sequence with period 2seventually.

Proof

Assume that \(A\geq B\). For any \(0\leq i\leq 2t-1\) and \(n\in { \mathbf{{N}}}_{0}\), we have

$$ A\leq x_{2(n+1)t+i}= \max \biggl\{ A, \frac{x_{2nt+i}}{x_{2(n+1)t-s+i}y_{2(n+1)t-s-t+i}} \biggr\} . $$
(2.6)

By Lemma 2.2 we may let \(\lim_{n\longrightarrow \infty }x_{2nt+i}=A_{i}\). Note that

$$ \lim_{n\longrightarrow \infty } \frac{x_{2nt+i}}{x_{2(n+1)t-s+i}y_{2(n+1)t-s-t+i}}\leq \lim _{n \longrightarrow \infty }\frac{x_{2nt+i}}{AB}=\frac{A_{i}}{AB}< A_{i} $$
(2.7)

and

$$ \lim_{n\longrightarrow \infty }x_{2(n+1)t+i}=A_{i}. $$
(2.8)

Thus we have \(x_{n}=A\) eventually. By Lemma 2.1, we see that \(y_{n}\) is a periodic sequence with period 2s eventually. The second case follows from the previously proved one by interchanging letters. The proof is complete. □

In the following, we assume \(AB=1\). For any \(i\in { \mathbf{{N}}}_{0}\), let

$$ \lim_{n\longrightarrow \infty } x_{2nt+i}=A_{i} \quad \mbox{if } A \geq B $$
(2.9)

and

$$ \lim_{n\longrightarrow \infty }y_{2nt+i}=B_{i}\quad \mbox{if } B \geq A. $$
(2.10)

Then \(A_{i}\geq A\) and \(B_{i}\geq B\).

Lemma 2.3

If\(A\geq B=1/A\)and\(A_{i}>A\)for some\(i\in { {\mathbf{{N}}}}_{0}\), then, for any\(k\in { {\mathbf{{N}}}}\), \(x_{2nt+ks+i}\)and\(y_{2nt-t+ks+i}\)are constant sequences eventually. If\(B\geq A=1/B\)and\(B_{i}>B\)for some\(i\in { {\mathbf{{N}}}}_{0}\), then, for any\(k\in { {\mathbf{{N}}}}\), \(y_{2nt+ks+i}\)and\(x_{2nt-t+ks+i}\)are constant sequences eventually.

Proof

Assume that \(A\geq B\) and \(A_{i}>A\) for some \(i\in { \mathbf{{N}}}_{0}\). Since \(A_{i}>A\), it follows from Lemma 2.2 and (1.1) that

$$ \begin{aligned}[b] x_{2nt+i}& =\max \biggl\{ A, \frac{x_{2(n-1)t+i}}{x_{2nt-s+i}y_{2nt-t-s+i}} \biggr\} \\ &=\frac{x_{2(n-1)t+i}}{x_{2nt-s+i}y_{2nt-t-s+i}} \quad \mbox{eventually}.\end{aligned} $$
(2.11)

From this we have

$$ \begin{aligned}[b] B&\leq \lim_{n\longrightarrow \infty }y_{2nt-t-s+i} \\ &=\lim_{n\longrightarrow \infty } \frac{x_{2(n-1)t+i}}{x_{2nt+i}x_{2nt-s+i}} \\ &=\frac{1}{A_{-s+i}}\leq \frac{1}{A}=B.\end{aligned} $$
(2.12)

This implies

$$ \lim_{n\longrightarrow \infty }x_{2nt-s+i}=A $$
(2.13)

and

$$ \lim_{n\longrightarrow \infty }y_{2nt-t-s+i}=B $$
(2.14)

and

$$ \lim_{n\longrightarrow \infty }y_{2nt-t+i}=\lim_{n\longrightarrow \infty }x_{2nt+i}x_{2nt-s+i}=A_{i}A. $$
(2.15)

Since

$$ x_{2nt+s+i} = \max \biggl\{ A, \frac{x_{2(n-1)t+s+i}}{x_{2nt+i}y_{2nt-t+i}} \biggr\} $$
(2.16)

and

$$ \lim_{n\longrightarrow \infty } \frac{x_{2(n-1)t+s+i}}{x_{2nt+i}y_{2nt-t+i}}= \frac{A_{s+i}}{A_{i}^{2}A}< A_{s+i}, $$
(2.17)

we see that \(x_{2nt+s+i} =A\) eventually. Note that

$$ \lim_{n\longrightarrow \infty }\frac{x_{2(n-1)t+s+i}}{y_{2nt-t+i}}= \frac{A}{AA_{i}}< B, $$
(2.18)

from which it follows that

$$ \begin{aligned}[b] y_{2nt-t+s+i} &= \max \biggl\{ B, \frac{x_{2(n-1)t+s+i}}{y_{2nt-t+i}} \biggr\} \\ &=B \quad \mbox{eventually},\end{aligned} $$
(2.19)

and

$$ \begin{aligned}[b] y_{2nt-t+2s+i} &= \max \biggl\{ B, \frac{x_{2(n-1)t+2s+i}}{y_{2nt-t+s+i}} \biggr\} \\ &=\frac{x_{2(n-1)t+2s+i}}{B}\quad \mbox{eventually}, \end{aligned} $$
(2.20)

and

$$ \begin{aligned}[b] x_{2nt+2s+i}& = \max \biggl\{ A, \frac{y_{2nt-t+2s+i}}{x_{2nt+s+i}} \biggr\} \\ &=\max \{A,x_{2(n-1)t+2s+i}\} \\ &=x_{2(n-1)t+2s+i} \quad \mbox{eventually}.\end{aligned} $$
(2.21)

If \(x_{2nt+2s+i}>A\) eventually, then, in a similar fashion, we obtain:

  1. (1)

    \(x_{2nt+3s+i} =A\) eventually and \(y_{2nt-t+3s+i} =B \) eventually.

  2. (2)

    \(x_{2nt+4s+i}\) and \(y_{2nt-t+4s+i}\) are constant sequences eventually.

If \(x_{2nt+2s+i}=A\) eventually, then \(y_{2nt-t+2s+i} =A/B\) eventually, and

$$ \begin{aligned}[b] y_{2nt-t+3s+i}& = \max \biggl\{ B, \frac{x_{2(n-1)t+3s+i}}{y_{2nt-t+2s+i}} \biggr\} \\ &=\max \biggl\{ B,\frac{x_{2(n-1)t+3s+i}B}{A} \biggr\} \\ &=\frac{x_{2(n-1)t+3s+i}B}{A}\quad \mbox{eventually},\end{aligned} $$
(2.22)

and

$$ \begin{aligned}[b] x_{2nt+3s+i}& = \max \biggl\{ A, \frac{y_{2nt-t+3s+i}}{x_{2nt+2s+i}} \biggr\} \\ &=\max \biggl\{ A,\frac{x_{2(n-1)t+3s+i}B}{A^{2}}\biggr\} \quad \mbox{eventually}.\end{aligned} $$
(2.23)

From this we see that if \(A=B\), then

$$ x_{2nt+3s+i}=x_{2(n-1)t+3s+i} \quad \mbox{eventually}, $$
(2.24)

and if \(A>B\), then

$$ x_{2nt+3s+i}=A\quad \mbox{ eventually}, $$
(2.25)

since

$$ \lim_{n\longrightarrow \infty }\frac{x_{2(n-1)t+3s+i}B}{A^{2}}= \frac{A_{3s+i}B}{A^{2}}< A_{3s+i}. $$
(2.26)

Using induction and arguments similar to the ones developed in the above given proof, we can show that, for any \(k\in { \mathbf{{N}}}\), \(x_{2nt+ks+i}\) and \(y_{2nt-t+ks+i}\) are constant sequences eventually. The second case follows from the previously proved one by interchanging letters. The proof is complete. □

Lemma 2.4

If\(A=1/B>B\)and for some\(i\in { {\mathbf{{N}}}}_{0}\), \(x_{2nt+i}>A\)eventually and\(A_{i}=A\), then, for any\(k\in { {\mathbf{{N}}}}\), \(x_{2nt+ks+i}\)and\(y_{2nt-t+ks+i}\)are constant sequences eventually. If\(B=1/A>A\)and for some\(i\in { {\mathbf{{N}}}}_{0}\), \(y_{2nt+i}>B\)eventually and\(B_{i}=B\), then, for any\(k\in { {\mathbf{{N}}}}\), \(y_{2nt+ks+i}\)and\(x_{2nt-t+ks+i}\)are constant sequences eventually.

Proof

Assume that \(A=1/B>B\) and for some \(i\in { \mathbf{{N}}}_{0}\), \(x_{2nt+i}>A\) eventually and \(A_{i}=A\). By (1.1) we have

$$ \begin{aligned}[b] x_{2nt+i}& = \max \biggl\{ A, \frac{y_{2nt-t+i}}{x_{2nt-s+i}} \biggr\} \\ &=\frac{y_{2nt-t+i}}{x_{2nt-s+i}} \quad \mbox{eventually}, \end{aligned} $$
(2.27)

and

$$ x_{2nt+s+i} = \max \biggl\{ A, \frac{x_{2(n-1)t+s+i}}{x_{2nt+i}y_{2nt-t+i}} \biggr\} $$
(2.28)

and

$$ \begin{aligned}[b] \lim_{n\longrightarrow \infty } \frac{x_{2(n-1)t+s+i}}{x_{2nt+i}y_{2nt-t+i}} &=\lim_{n \longrightarrow \infty } \frac{x_{2(n-1)t+s+i}}{x^{2}_{2nt+i}x_{2nt-s+i}} \\ &\leq \frac{A_{s+i}}{A^{3}}< A_{s+i}. \end{aligned} $$
(2.29)

Then we see that \(x_{2nt+s+i} =A\) eventually. From this and \(y_{2nt-t+i}\geq A^{2}\) eventually it follows that

$$ \begin{aligned}[b] y_{2nt-t+s+i}& = \max \biggl\{ B, \frac{x_{2(n-1)t+s+i}}{y_{2nt-t+i}} \biggr\} \\ &=\max \biggl\{ B,\frac{A}{y_{2nt-t+i}} \biggr\} \\ &=B \quad \mbox{eventually}, \end{aligned} $$
(2.30)

and

$$ \begin{aligned}[b] y_{2nt-t+2s+i} &= \max \biggl\{ B, \frac{x_{2(n-1)t+2s+i}}{y_{2nt-t+s+i}} \biggr\} \\ &=\frac{x_{2(n-1)t+2s+i}}{B} \quad \mbox{eventually},\end{aligned} $$
(2.31)

and

$$ \begin{aligned}[b] x_{2nt+2s+i}& = \max \biggl\{ A, \frac{y_{2nt-t+2s+i}}{x_{2nt+s+i}} \biggr\} \\ &=\max \{A,x_{2(n-1)t+2s+i}\} \\ &=x_{2(n-1)t+2s+i} \quad \mbox{eventually}.\end{aligned} $$
(2.32)

Thus \(x_{2nt+2s+i}\) and \(y_{2nt-t+2s+i}\) are constant sequences eventually. Using arguments similar to the ones developed in the proof of Lemma 2.3, we can show that, for any \(k\in { \mathbf{{N}}}\), \(x_{2nt+ks+i}\) and \(y_{2nt-t+ks+i}\) are constant sequences eventually. The second case follows from the previously proved one by interchanging letters. The proof is complete. □

Theorem 2.2

  1. (1)

    Assume\(A=1/B>B\). Then one of the following statements holds.

    1. (i)

      \(x_{n}=A\)eventually and\(y_{n}\)is a periodic sequence with period 2seventually.

    2. (ii)

      Ifsis odd, then\(x_{n}\), \(y_{n}\)are periodic sequences with period 2teventually.

    3. (iii)

      Ifsis even, then\(x_{n}\)is a periodic sequence with period 2teventually and\(y_{n}\)is a periodic sequence with period\(2st\)eventually.

  2. (2)

    Assume\(B=1/A>A\). Then one of the following statements holds.

    1. (i)

      \(y_{n}=B\)eventually and\(x_{n}\)is a periodic sequence with period 2seventually.

    2. (ii)

      Ifsis odd, then\(x_{n}\), \(y_{n}\)are periodic sequences with period 2teventually.

    3. (iii)

      Ifsis even, then\(y_{n}\)is a periodic sequence with period 2teventually and\(x_{n}\)is a periodic sequence with period\(2st\)eventually.

Proof

Assume that \(A=1/B>B\). If \(x_{n}=A\) eventually, then by Lemma 2.1 we see that \(y_{n}\) is a periodic sequence with period 2s eventually. Now we assume that \(x_{n}\neq A\) eventually. Then we have \(A_{i}>A\) (or \(x_{2nt+i}>A\) eventually and \(A_{i}=A\)) for some \(0\leq i\leq 2t-1\).

If s is odd, then \(\gcd (2t,s)=1\). Thus, for every \(j\in \{0,1,2,\ldots ,2t-1\} \), there exist some \(1\leq i_{j}\leq 2t\) and integer \(\lambda _{j}\) such that \(i_{j}s=\lambda _{j} 2t+j\) since \(\{rs:0\leq r\leq 2t-1\}=\{0,1,2,\ldots ,2t-1\}\) (mod 2t). By Lemma 2.3 and Lemma 2.4 we see that, for any \(k\in { \mathbf{{N}}}\), \(x_{2nt+ks+i}\) and \(y_{2nt-t+ks+i}\) are constant sequences eventually. Thus, for any \(0\leq r\leq 2t-1\), \(x_{2nt+r}\) and \(y_{2nt+r}\) are constant sequences eventually, which implies that \(x_{n}\), \(y_{n}\) are periodic sequences with period 2t eventually.

In the following, we assume that s is even with \(s=2s'\). Then \(\gcd (t,s')=1\) and t is odd. Thus, for every \(j\in \{0,1,2,\ldots ,t-1\} \), there exist some \(1\leq i_{j}\leq t\) and integer \(\lambda _{j}\) such that \(i_{j}s'=\lambda _{j} t+j\) and \(i_{j}s=\lambda _{j} 2t+2j\).

If \(x_{2nt+i}\neq A\) eventually for some \(i\in \{0,2,\ldots \}\) and \(x_{2nt+l}\neq A\) eventually for some \(l\in \{1,3,\ldots \}\), then by Lemma 2.3 and Lemma 2.4 we see that, for any \(k\in { \mathbf{{N}}}\), \(x_{2nt+ks+i}\), \(y_{2nt-t+ks+i}\), \(x_{2nt+ks+l}\) and \(y_{2nt-t+ks+l}\) are constant sequences eventually. Thus, for any \(0\leq r\leq 2t-1\), \(x_{2nt+r}\) and \(y_{2nt+r}\) are constant sequences eventually, which implies that \(x_{n}\), \(y_{n}\) are periodic sequences with period 2t eventually.

If \(x_{2nt+i}\neq A\) eventually for some \(i\in \{0,2,\ldots \}\) and \(x_{2nt+l}=A\) eventually for any \(l\in \{1,3,\ldots \}\), then by Lemma 2.3 and Lemma 2.4 we see that, for any \(k\in { \mathbf{{N}}}\), \(x_{2nt+ks+i}\) and \(y_{2nt-t+ks+i}\) are constant sequences eventually. This implies that, for every \(r\in \{0,1,2,\ldots ,2t-1\}\), \(x_{2nt+r}\) is constant sequence eventually and for every \(l\in \{1,3,\ldots \}\), \(y_{2nst+l}\) is constant sequence eventually. By (1.1) we see that there exists \(N\in { \mathbf{{N}}}\) such that, for any \(n\geq N\) and \(r\in \{0,2,\ldots \}\),

$$ y_{2nt+r} = \max \biggl\{ B ,\frac{A}{y_{2nt+r-s}} \biggr\} . $$
(2.33)

Then we have \(y_{2nt+r}y_{2nt+r-s}\geq A\). Thus, for any \(n\geq N\) and \(l\in \{1,3,\ldots \}\) and \(k\in { \mathbf{{N}}}\),

$$\begin{aligned} B&\leq y_{2nt+t+2ks+l} = \max \biggl\{ B , \frac{x_{2nt+2ks+l}}{y_{2nt+t+2ks+l-s}} \biggr\} \\ &= \max \biggl\{ B , \frac{Ay_{2nt+t+2ks+l-2s}}{y_{2nt+t+2ks+l-s}y_{2nt+t+2ks+l-2s}} \biggr\} \\ &\leq \max \{B,y_{2nt+t+2ks-2s+l}\} \\ &= y_{2nt+t+2ks-2s+l}\quad \mbox{eventually}. \end{aligned}$$
(2.34)

Then, for every \(n\geq N\) and \(l\in \{1,3,\ldots \}\), we have

$$ B\leq \cdots \leq y_{2nt+t+2ks+l}\leq y_{2nt+t+2ks-2s+l}\leq y_{2nt+t+2s+l} \leq y_{2nt+t+l}. $$
(2.35)

We claim that, for every \(n\geq N\) and \(l\in \{1,3,\ldots \}\), \(\{y_{2nt+t+2ks+l}\}_{k\in { \mathbf{{N}}}}\) is a constant sequence eventually. Assume on the contrary that, for some \(n\geq N\) and some \(l\in \{1,3,\ldots \}\), \(\{y_{2nt+t+2ks+l}\}_{k\in { \mathbf{{N}}}}\) is not a constant sequence eventually. Then there exists a sequence of positive integers \(k_{1}< k_{2}<\cdots \) such that, for any \(r\in { \mathbf{{N}}}\), we have

$$ \begin{aligned}[b] B&< y_{2nt+t+2k_{r}s+l} = \frac{A}{y_{2nt+t+2k_{r}s+l-s}} \\ &< y_{2nt+t+2k_{r-1}s+l}=\frac{A}{ y_{2nt+t+2k_{r-1}s+l-s}}, \end{aligned} $$
(2.36)

which implies \(y_{2nt+t+2k_{r}s+l-s}>y_{2nt+t+2k_{r-1}s+l-s}\) for any \(r\in { \mathbf{{N}}}\). This is a contradiction. Take \(ps>N\). Then \(y_{2nst+t+l}=y_{2pst+t+2(nt-pt)s+l}\) is a constant sequence eventually for any \(l\in \{1,3,\ldots \}\). From the above we see that \(y_{n}\) is a periodic sequence with period \(2st\) eventually.

In a similar fashion, we can show that if \(x_{2nt+i}=A\) eventually for any \(i\in \{0,2,\ldots \}\) and \(x_{2nt+l}\neq A\) eventually for some \(l\in \{1,3,\ldots \}\), then also statement (1(iii)) holds.

The second case follows from the previously proved one by interchanging letters. The proof is complete. □

Now we assume that \(A=B=1\). Then, for any \(0\leq i\leq 2t-1\) and \(n\in { \mathbf{{N}}}_{0}\), we have \(1\leq x_{2(n+1)t+i}\leq x_{2nt+i}\) eventually and \(1\leq y_{2(n+1)t+i}\leq y_{2nt+i}\) eventually.

Lemma 2.5

Let\(A=B=1\)and\(s\geq t\). Then the following statements hold.

  1. (1)

    If\(A_{i}=1\), then\(B_{t+i}=1\). If\(B_{i}=1\), then\(A_{t+i}=1\).

  2. (2)

    If\(x_{N}=1\)for some\(N\in { {\mathbf{{N}}}}\)and\(A_{2nt+N+ks}=1\)for any\(k,n\in { {\mathbf{{N}}}}\), then\(x_{2nt+N+ks}=y_{2nt+t+ks+N}=1\)for any\(k,n\in { {\mathbf{{N}}}}\). If\(y_{N}=1\)for some\(N\in { {\mathbf{{N}}}}\)and\(B_{2nt+N+ks}=1\)for any\(k,n\in { {\mathbf{{N}}}}\), then\(y_{2nt+N+ks}=x_{2nt+t+ks+N}=1\)for any\(k,n\in { {\mathbf{{N}}}}\).

  3. (3)

    Ifsis even and\(\gcd (s,t)=1\), then\(1\in \{x_{n}:n\in \{0,2,\ldots \}\}\cup \{y_{t+n}:n\in \{0,2,\ldots \}\}\)and\(1\in \{x_{n}:n\in \{1,3,\ldots \}\}\cup \{y_{t+n}:n\in \{1,3,\ldots \}\}\).

  4. (4)

    \(1\in \{x_{n}:n\in { {\mathbf{{N}}}}\}\cup \{y_{n}:n\in { {\mathbf{{N}}}}\}\).

Proof

(1) Assume that \(A_{i}=1\). Assume on the contrary that \(B_{t+i}>1\). It follows from (1.1) that

$$ y_{2nt+t+i} =\frac{x_{2nt+i}}{y_{2nt+t-s+i}}. $$
(2.37)

This implies

$$ 1\leq \lim_{n\longrightarrow \infty }y_{2nt+t-s+i}=\frac{1}{B_{t+i}}< 1. $$
(2.38)

This is a contradiction. The second case follows from the previously proved one by interchanging letters.

(2) If \(x_{N}=1\) for some \(N\in { \mathbf{{N}}}\), then \(x_{2nt+N}=1\) for any \(n\in { \mathbf{{N}}}\). It follows from (1.1) that

$$ \begin{aligned}[b] y_{2nt+t+N}& = \max \biggl\{ 1, \frac{x_{2nt+N}}{y_{2nt+t-s+N}} \biggr\} \\ &=\max \biggl\{ 1,\frac{1}{y_{2nt+t-s+N}}\biggr\} =1\end{aligned} $$
(2.39)

and

$$ \begin{aligned}[b] y_{2nt+t+s+N}& = \max \biggl\{ 1, \frac{x_{2nt+s+N}}{y_{2nt+t+N}} \biggr\} \\ &=x_{2nt+s+N}\end{aligned} $$
(2.40)

and

$$ \begin{aligned}[b] x_{2(n+1)t+N+s}& = \max \biggl\{ 1, \frac{y_{2nt+t+s+N}}{x_{2(n+1)t+N}} \biggr\} \\ &=\max \{1,y_{2nt+t+s+N}\} \\ &=y_{2nt+t+s+N} \\ &=x_{2nt+N+s}.\end{aligned} $$
(2.41)

Thus \(x_{2nt+N+s}=y_{2nt+t+s+N}=1\) for any \(n\in { \mathbf{{N}}}\). In a similar fashion, we can show that \(x_{2nt+N+ks}=y_{2nt+t+ks+N}=1\) for any \(k,n\in { \mathbf{{N}}}\). The second case follows from the previously proved one by interchanging letters.

(3) If s is even and \(\gcd (s,t)=1\), then t is odd. Assume on the contrary that \(1\notin \{x_{n}:n\in \{0,2,\ldots \}\}\cup \{y_{t+n}:n\in \{0,2, \ldots \}\}\). Then it follows from (1.1) that, for any \(n\in { \mathbf{{N}}}\),

$$ \begin{aligned}[b] y_{2nt+t}& = \max \biggl\{ 1, \frac{x_{2nt}}{y_{2nt+t-s}} \biggr\} \\ &=\frac{x_{2nt}}{y_{2nt+t-s}}>1 \end{aligned} $$
(2.42)

and

$$ \begin{aligned}[b] x_{2nt+2t-s}& = \max \biggl\{ 1, \frac{y_{2nt+t-s}}{x_{2nt+2t-2s}} \biggr\} \\ &=\frac{y_{2nt+t-s}}{x_{2nt+2t-2s}}>1.\end{aligned} $$
(2.43)

Thus

$$ \begin{aligned}[b] x_{2nt}& > x_{2nt-2(s-t)} \\ &>x_{2nt-4(s-t)} \\ &\quad \cdots \\ &>x_{2t(n-t+s)}. \end{aligned} $$
(2.44)

This is a contradiction.

(4) Case (4) is treated similarly to case (3). The proof is complete. □

Theorem 2.3

Let\(A=B=1\)and\(s\geq t\). Then one of the following statements holds.

  1. (1)

    \(x_{n}=1\)eventually and\(y_{n}\)is a periodic sequence with period 2seventually.

  2. (2)

    \(y_{n}=1\)eventually and\(x_{n}\)is a periodic sequence with period 2seventually.

  3. (3)

    \(x_{n}\), \(y_{n}\)are periodic sequences with period 2teventually.

Proof

If \(x_{n}=1\) (or \(y_{n}=1\)) eventually, then by Lemma 2.1 we see that \(y_{n}\) (or \(x_{n}\)) is a periodic sequence with period 2s eventually. Now we assume that \(x_{n}\neq1\) eventually. Then we have \(A_{i}>1\) for some \(0\leq i\leq 2t-1\) or \(\lim_{n\longrightarrow \infty }x_{n}=1\).

If s is odd, then \(\gcd (2t,s)=1\). Thus, for every \(j\in \{0,1,2,\ldots ,2t-1\} \), there exist some \(1\leq i_{j}\leq 2t\) and integer \(\lambda _{j}\) such that \(i_{j}s=\lambda _{j} 2t+j\). By Lemma 2.3 and Lemma 2.5 we see that, for any \(k\in { \mathbf{{N}}}\), \(x_{2nt+ks+i}\) and \(y_{2nt-t+ks+i}\) are constant sequences eventually, or for some \(N\in { \mathbf{{N}}}\), \(x_{2nt+N+ks}=y_{2nt+t+ks+N}=1\) for any \(k,n\in { \mathbf{{N}}}\), or for some \(N\in { \mathbf{{N}}}\), \(y_{2nt+N+ks}=x_{2nt+t+ks+N}=1\) for any \(k,n\in { \mathbf{{N}}}\). Thus, for any \(0\leq r\leq 2t-1\), \(x_{2nt+r}\) and \(y_{2nt+r}\) are constant sequences eventually, which implies that \(x_{n}\), \(y_{n}\) are periodic sequences with period 2t eventually.

In the following, we assume that s is even with \(s=2s'\), then \(\gcd (t,s')=1\) and t is odd. Thus, for every \(j\in \{0,1,2,\ldots ,t-1\} \), there exist some \(1\leq i_{j}\leq t\) and integer \(\lambda _{j}\) such that \(i_{j}s=\lambda _{j} 2t+2j\).

If \(A_{i}>1\) for some \(i\in \{0,2,\ldots \}\), then by Lemma 2.3 we see that, for any \(k\in { \mathbf{{N}}}\), \(x_{2nt+ks+i}\) and \(y_{2nt-t+ks+i}\) are constant sequences eventually. If \(A_{i}=1\) for any \(i\in \{0,2,\ldots \}\), then by Lemma 2.5 we have \(B_{t+i}=1\) for any \(i\in \{0,2,\ldots \}\) and \(x_{2nt+i+ks}=y_{2nt-t+ks+i}=1\) for any \(k\in { \mathbf{{N}}}\) eventually. In a similar fashion, also we can show that, for any \(i\in \{1,3,\ldots \}\), \(x_{2nt+ks+i}\) and \(y_{2nt-t+ks+i}\) are constant sequences eventually for any \(k\in { \mathbf{{N}}}\), or \(x_{2nt+i+ks}=y_{2nt-t+ks+i}=1\) for any \(k\in { \mathbf{{N}}}\) eventually for any \(i\in \{1,3,\ldots \}\) and \(k\in { \mathbf{{N}}}\). Thus, for any \(0\leq r\leq 2t-1\), \(x_{2nt+r}\) and \(y_{2nt+r}\) are constant sequences eventually. This implies that \(x_{n}\), \(y_{n}\) are periodic sequences with period 2t eventually.

Using the previously proved one by interchanging letters, also we can show that if \(y_{n}\neq1\) eventually, then \(x_{n}\), \(y_{n}\) are periodic sequences with period 2t eventually. The proof is complete. □

In Example 3.1 of [37], we showed that the equation

$$ x_{n}=\frac{x_{n-t}}{x_{n-s}} (t>s) $$
(2.45)

has a positive solution \(z_{n}\) (\(n\geq -t\)) with \(1< z_{n+1}< z_{n}\) for any \(n\geq -t\) and \(\lim_{n\longrightarrow \infty }z_{n}=1\).

From Example 3.1 of [37], we obtain the following theorem.

Theorem 2.4

Let\(A\leq 1\)and\(B\leq 1\)and\(s< t\). Assume\(z_{n}\) (\(n\geq -t\)) is a positive solution of (2.45) with\(1< z_{n+1}< z_{n}\)for any\(n\geq -t\)and\(\lim_{n\longrightarrow \infty }z_{n}=1\). Then equation (1.1) have a solution\((x_{n},y_{n})\)with\(1< x_{n+1}=y_{n+1}=z_{n+1}< x_{n}=y_{n}=z_{n}\)for any\(n\geq -t\)and\(\lim_{n\longrightarrow \infty }x_{n}=\lim_{n\longrightarrow \infty }y_{n}=1\).

References

  1. Berenhaut, K.S., Foley, J.D., Stević, S.: Boundedness character of positive solutions of a max difference equation. J. Differ. Equ. Appl. 12, 1193–1199 (2006)

    Article  MathSciNet  Google Scholar 

  2. Cranston, D.M., Kent, C.M.: On the boundedness of positive solutions of the reciprocal max-type difference equation \(x_{n} =\max \{A^{1}_{n-1}/x_{n-1}, A^{2}_{n-1}/x_{n-2}, \ldots , A^{t}_{n-1}/x_{n-t} \}\) with periodic parameters. Appl. Math. Comput. 221, 144–151 (2013)

    MathSciNet  Google Scholar 

  3. Elsayed, E.M., Iričanin, B.D.: On a max-type and a min-type difference equation. Appl. Math. Comput. 215, 608–614 (2009)

    MathSciNet  MATH  Google Scholar 

  4. Elsayed, E.M., Iričanin, B.D., Stević, S.: On the max-type equation \(x_{n+1} =\max \{A_{n}/x_{n}, x_{n-1}\}\). Ars Comb. 95, 187–192 (2010)

    MATH  Google Scholar 

  5. Fotiades, E., Papaschinopoulos, G.: On a system of difference equations with maximum. Appl. Math. Comput. 221, 684–690 (2013)

    MathSciNet  MATH  Google Scholar 

  6. Iričanin, B.D., Stević, S.: Eventually constant solutions of a rational difference equation. Appl. Math. Comput. 215, 854–856 (2009)

    MathSciNet  MATH  Google Scholar 

  7. Iričanin, B.D., Stević, S.: Global attractivity of the max-type difference equation \(x_{n}=\max \{c, x^{p}_{n-1}/\prod_{j=2}^{k}x_{n-j}^{p_{j}}\}\). Util. Math. 91, 301–304 (2013)

    MathSciNet  MATH  Google Scholar 

  8. Liu, W., Stević, S.: Global attractivity of a family of nonautonomous max-type difference equations. Appl. Math. Comput. 218, 6297–6303 (2012)

    MathSciNet  MATH  Google Scholar 

  9. Liu, W., Yang, X., Stević, S.: On a class of nonautonomous max-type difference equations. Abstr. Appl. Anal. 2011, Article ID 327432 (2011)

    MathSciNet  Google Scholar 

  10. Papaschinopoulos, G., Schinas, J.: On a system of two nonlinear difference equations. J. Math. Anal. Appl. 219, 415–426 (1998)

    Article  MathSciNet  Google Scholar 

  11. Papaschinopoulos, G., Schinas, J., Hatzifilippidis, V.: Global behavior of the solutions of a max-equation and of a system of two max-equations. J. Comput. Anal. Appl. 5, 237–254 (2003)

    MathSciNet  MATH  Google Scholar 

  12. Qin, B., Sun, T., Xi, H.: Dynamics of the max-type difference equation \(x_{n+1} = \max \{A/x_{n}, x_{n-k}\}\). J. Comput. Anal. Appl. 14, 856–861 (2012)

    MathSciNet  MATH  Google Scholar 

  13. Sauer, T.: Global convergence of max-type equations. J. Differ. Equ. Appl. 17, 1–8 (2011)

    Article  MathSciNet  Google Scholar 

  14. Shi, Q., Su, X., Yuan, G.: Characters of the solutions to a generalized nonlinear max-type difference equation. Chin. Q. J. Math. 28, 284–289 (2013)

    MATH  Google Scholar 

  15. Stefanidou, G., Papaschinopoulos, G., Schinas, C.: On a system of max difference equations. Dyn. Contin. Discrete Impuls. Syst., Ser. A Math. Anal. 14, 885–903 (2007)

    MathSciNet  MATH  Google Scholar 

  16. Stević, S.: On the recursive sequence \(x_{n+1}=\max \{c, x^{p}_{n}/x_{n-1}^{p}\}\). Appl. Math. Lett. 21, 791–796 (2008)

    Article  MathSciNet  Google Scholar 

  17. Stević, S.: Boundedness character of a class of difference equations. Nonlinear Anal. TMA 70, 839–848 (2009)

    Article  MathSciNet  Google Scholar 

  18. Stević, S.: Global stability of a difference equation with maximum. Appl. Math. Comput. 210, 525–529 (2009)

    MathSciNet  MATH  Google Scholar 

  19. Stević, S.: Global stability of a max-type difference equation. Appl. Math. Comput. 216, 354–356 (2010)

    MathSciNet  MATH  Google Scholar 

  20. Stević, S.: On a generalized max-type difference equation from automatic control theory. Nonlinear Anal. TMA 72, 1841–1849 (2010)

    Article  MathSciNet  Google Scholar 

  21. Stević, S.: Periodicity of max difference equations. Util. Math. 83, 69–71 (2010)

    MathSciNet  MATH  Google Scholar 

  22. Stević, S.: Periodicity of a class of nonautonomous max-type difference equations. Appl. Math. Comput. 217, 9562–9566 (2011)

    MathSciNet  MATH  Google Scholar 

  23. Stević, S.: Solutions of a max-type system of difference equations. Appl. Math. Comput. 218, 9825–9830 (2012)

    MathSciNet  MATH  Google Scholar 

  24. Stević, S.: On some periodic systems of max-type difference equations. Appl. Math. Comput. 218, 11483–11487 (2012)

    MathSciNet  MATH  Google Scholar 

  25. Stević, S.: On a symmetric system of max-type difference equations. Appl. Math. Comput. 219, 8407–8412 (2013)

    MathSciNet  MATH  Google Scholar 

  26. Stević, S.: Product-type system of difference equations of second-order solvable in closed form. Electron. J. Qual. Theory Differ. Equ. 2014, 56 (2014)

    Article  MathSciNet  Google Scholar 

  27. Stević, S.: Representation of solutions of bilinear difference equations in terms of generalized Fibonacci sequences. Electron. J. Qual. Theory Differ. Equ. 2014, 67 (2014)

    Article  MathSciNet  Google Scholar 

  28. Stević, S., Alghamdi, M.A., Alotaibi, A.: Long-term behavior of positive solutions of a system of max-type difference equations. Appl. Math. Comput. 235, 567–574 (2014)

    MathSciNet  MATH  Google Scholar 

  29. Stević, S., Alghamdi, M.A., Alotaibi, A., Shahzad, N.: Eventual periodicity of some systems of max-type difference equations. Appl. Math. Comput. 236, 635–641 (2014)

    MathSciNet  MATH  Google Scholar 

  30. Stević, S., Alghamdi, M.A., Alotaibi, A., Shahzad, N.: Boundedness character of a max-type system of difference equations of second order. Electron. J. Qual. Theory Differ. Equ. 2014, 45 (2014)

    Article  MathSciNet  Google Scholar 

  31. Stević, S., Iričanin, B.D.: On a max-type difference inequality and its applications. Discrete Dyn. Nat. Soc. 2010, Article ID 975740 (2010)

    Article  MathSciNet  Google Scholar 

  32. Sun, T., He, Q., Wu, X., Xi, H.: Global behavior of the max-type difference equation \(x_{n} =\max \{1/ x_{n-m},A_{n}/ x_{n-r}\}\). Appl. Math. Comput. 248, 687–692 (2014)

    MathSciNet  MATH  Google Scholar 

  33. Sun, T., Liu, J., He, Q., Liu, X.: Eventually periodic solutions of a max-type difference equation. Sci. World J. 2014, Article ID 219437 (2014)

    Google Scholar 

  34. Sun, T., Qin, B., Xi, H., Han, C.: Global behavior of the max-type difference equation \(x_{n+1}=\max \{1/x_{n}, A_{n}/x_{n-1}\}\). Abstr. Appl. Anal. 2009, Article ID 152964 (2009)

    Article  Google Scholar 

  35. Sun, T., Xi, H.: On the solutions of a system of difference equations with maximum. Appl. Math. Comput. 290, 292–297 (2016)

    MathSciNet  MATH  Google Scholar 

  36. Sun, T., Xi, H., Han, C., Qin, B.: Dynamics of the max-type difference equation \(x_{n} =\max \{ 1/ x_{n-m} , A_{n}/ x_{n-r}\}\). J. Appl. Math. Comput. 38, 173–180 (2012)

    Article  MathSciNet  Google Scholar 

  37. Sun, T., Xi, H., Quan, W.: Existence of monotone solutions of a difference equation. Discrete Dyn. Nat. Soc. 2008, Article ID 917560 (2008)

    MathSciNet  MATH  Google Scholar 

  38. Xiao, Q., Shi, Q.: Eventually periodic solutions of a max-type equation. Math. Comput. Model. 57, 992–996 (2013)

    Article  MathSciNet  Google Scholar 

  39. Yazlik, Y., Tollu, D.T., Taskara, N.: On the solutions of a max-type difference equation system. Math. Methods Appl. Sci. 38, 4388–4410 (2015)

    Article  MathSciNet  Google Scholar 

Download references

Acknowledgements

The authors would like to thank the referees for their valuable comments and suggestions.

Availability of data and materials

None.

Funding

The research was supported by NNSF of China (11761011, 71862003) and NSF of Guangxi (2018GXNSFAA294010) and SF of Guangxi University of Finance and Economics (2019QNB10).

Author information

Authors and Affiliations

Authors

Contributions

All authors participated in every phase of research conducted for this paper. All authors read and approved the final manuscript.

Corresponding author

Correspondence to Caihong Han.

Ethics declarations

Competing interests

The authors declare that they have no competing interests.

Rights and permissions

Open Access This article is licensed under a Creative Commons Attribution 4.0 International License, which permits use, sharing, adaptation, distribution and reproduction in any medium or format, as long as you give appropriate credit to the original author(s) and the source, provide a link to the Creative Commons licence, and indicate if changes were made. The images or other third party material in this article are included in the article’s Creative Commons licence, unless indicated otherwise in a credit line to the material. If material is not included in the article’s Creative Commons licence and your intended use is not permitted by statutory regulation or exceeds the permitted use, you will need to obtain permission directly from the copyright holder. To view a copy of this licence, visit http://creativecommons.org/licenses/by/4.0/.

Reprints and permissions

About this article

Check for updates. Verify currency and authenticity via CrossMark

Cite this article

Su, G., Han, C., Sun, T. et al. On the solutions of a max-type system of difference equations of higher order. Adv Differ Equ 2020, 213 (2020). https://doi.org/10.1186/s13662-020-02673-2

Download citation

  • Received:

  • Accepted:

  • Published:

  • DOI: https://doi.org/10.1186/s13662-020-02673-2

MSC

Keywords